2.5 Критерий Коши

Если для исследования сходимости последовательности применять определение предела, то мы заранее должны знать, является ли данная последовательность сходящейся и значение ее предела. Используя определение предела, мы можем лишь доказывать выдвинутую гипотезу. Однако в ряде случаев по самому виду последовательности трудно определить, является ли она сходящейся или расходящейся. Например, $x_n = 1 + \frac{1}{2} + \ldots + \frac{1}{n}$ . В связи с этим возникает необходимость найти внутреннее свойство последовательности, равносильное сходимости и не
зависящее от числа $a$ – предела последовательности. Мы докажем, что таким свойством является фундаментальность.

Определение. Последовательность $\{x_n\}$ называется фундаментальной (сходящейся в себе), если для любого $\varepsilon > 0$ найдется такой номер $N$, зависящий, вообще говоря, от $\varepsilon$, что для всех номеров $n \geqslant N$, $m \geqslant N$ справедливо неравенство $|x_n — x_m| < \varepsilon$.

Существенное отличие определения фундаментальности от определения предела состоит в том, что в определении предела мы должны знать значение предела, а в определении фундаментальности это не требуется. Смысл определения предела состоит в том, что все элементы последовательности с достаточно большими номерами мало отличаются от значения предела, т. е. $|x_n — a| < \varepsilon$ при $n \geqslant N = N(\varepsilon)$. В определении фундаментальности требуется чтобы все элементы последовательности с достаточно большими номерами мало отличались друг от друга $\Big(|x_n — x_m| < \varepsilon$, $n, m \geqslant N = N(\varepsilon)\Big).$

Равносильность сходимости последовательности и ее фундаментальности устанавливает следующая теорема.

Теорема (критерий Коши). Для того чтобы последовательность была сходящейся, необходимо и достаточно, чтобы она была фундаментальной.

Необходимость доказывается совсем просто. В самом деле, нужно показать, что из сходимости следует фундаментальность. Пусть последовательность $\{x_n\}$ сходится и $\lim\limits_{n\to \infty}x_n = a$. Зададим $\varepsilon > 0$ и найдем номер $N$, такой, что для любого $n \geqslant N$ справедливо неравенство $|x_n — a| < \frac{\varepsilon}{2}$. Если $n, m \geqslant N$, то получим $$|x_n — x_m| \leqslant |x_n — a| + |x_m — a| < \frac{\varepsilon}{2} + \frac{\varepsilon}{2} = \varepsilon$$ а это и означает, что $\{x_n\}$ – фундаментальна.

Достаточность. Нужно показать, что из фундаментальности последовательности следует ее сходимость. Сначала мы покажем, что из фундаментальности следует ограниченность. Затем, используя лемму Больцано – Вейерштрасса, из ограниченной последовательности выделим сходящуюся подпоследовательность и, наконец, снова используя фундаментальность, покажем, что и вся последовательность сходится к тому же пределу, что и выделенная подпоследовательность.

Итак, пусть $\{x_n\}$ – фундаментальная последовательность. Докажем ее ограниченность. Зададим $\varepsilon = 1$ и, пользуясь фундаментальностью, найдем номер $N_1$, такой, что для любых $n, m \geqslant N_1$ справедливо неравенство $|x_n — x_m| < 1$. Зафиксируем $m = N_1$. Тогда получим, что для всех $n \geqslant N_1$ имеет место неравенство $|x_n — x_m| < 1$, т. е. ${x_N}_1 — 1 < x_n < {x_N}_1 + 1$. Отсюда следует, что $|x_n| \leqslant |{x_N}_1| + 1$ для всех $n \geqslant N_1$. Во множестве $E = \{|{x_N}_1| + 1, |x_1| , \ldots , |{x_N}_1 − 1|\}$, состоящего из конечного числа элементов, выберем наибольший $A = \max\{|{x_N}_1| + 1, |x_1| ,\ldots, |{x_N}_1 − 1|\}$. Тогда получим, что $|x_n| \leqslant A$ для всех $n = 1, 2,\ldots$, а это и означает, что $\{x_n\}$ – ограниченная последовательность.

Применяя теперь к ограниченной последовательности $\{x_n\}$ лемму Больцано – Вейерштрасса, выделим из нее сходящуюся подпоследовательность ${\{{x_n}_k\}}^\infty_{k = 1}$ и обозначим через a предел этой подпоследовательности. Покажем, что вся последовательность $\{x_n\}$ также сходится к числу a, т. е. что $\lim\limits_{n\to \infty}x_n = a$.

Зададим $\varepsilon > 0$ и, пользуясь фундаментальностью последовательности $\{x_n\}$, найдем такой номер $N$, что для всех номеров $n, m \geqslant N$ справедливо неравенство $|x_n − x_m| < \frac{\varepsilon}{2}$. Далее, пользуясь тем, что $\lim\limits_{k\to \infty}{x_n}_k = a$, для заданного $\varepsilon$ найдем номер $k$, такой, что $n_k \geqslant N$ (это возможно, поскольку $n_k \rightarrow \infty$ при $k \rightarrow \infty$) и $|{x_n}_k — a| < \frac{\varepsilon}{2}$. Положим $m = n_k$. Тогда получим, что для любого $n \geqslant N$ справедливо неравенство $|x_n − {x_n}_k| < \frac{\varepsilon}{2}$. Отсюда следует, что для $n \geqslant N$ $$|x_n — a| \leqslant |x_n — {x_n}_k| + |{x_n}_k — a| < \frac{\varepsilon}{2} + \frac{\varepsilon}{2} = \varepsilon.$$

Итак, для заданного $\varepsilon > 0$ мы нашли номер $N$, начиная с которого справедливо неравенство $|x_n — a| < \varepsilon$. Поскольку выбранное $\varepsilon > 0$ произвольно, то по определению предела последовательности получаем, что $\lim\limits_{n\to \infty}x_n = a$.

Определение фундаментальности последовательности можно сформулировать в такой эквивалентной форме.

Определение. Последовательность $\{x_n\}$ называется фундаментальной, если для любого $\varepsilon > 0$ найдется такой номер $N$, зависящий, вообще говоря, от $\varepsilon$, что для любого $n \geqslant N$ и для любого $p \in N$ справедливо неравенство $|x_{n + p} — x_n| < \varepsilon$.

Пользуясь этим определением, скажем, что последовательность $\{x_n\}$ не является фундаментальной, если найдется такое $\varepsilon_0 > 0$, что для любого $N$ существуют такой номер $n \geqslant N$ и такое натуральное число $p$, что $|x_{n + p} − x_n| \geqslant \varepsilon_0$.

Пример 1. Рассмотрим последовательность $x_n = 1 + \frac{1}{2} + \ldots + \frac{1}{n}$. Для натуральных $n$ и $p$ имеем $x_{n + p} − x_n = \frac{1}{n + 1} + \ldots + \frac{1}{n + p} \geqslant \frac{1}{n + p} + \ldots + \frac{1}{n + p} = \frac{p}{n + p}$. Если $n$ зафиксировано, то для $p = n$ получаем $|x_{n + p} − x_n| \geqslant \frac{1}{2}$. Выберем $\varepsilon_0 = \frac{1}{2} > 0$. Тогда для любого номера $N$ положим $n = N$, $p = n$ и будем иметь $|x_{n + p} − x_n| \geqslant \varepsilon_0$. Это означает, что данная последовательность не является фундаментальной и, следовательно, в силу критерия Коши, она расходится.

Пример 2. Покажем, что последовательность $x_n = \frac{\sin 1}{1^2} + \frac{\sin 2}{2^2} + \ldots + \frac{\sin n}{n^2}$ фундаментальна, а значит, сходящаяся. Для натуральных $n$ и $p$ имеем $$|x_{n + p} − x_n| \leqslant \frac{1}{(n + 1)^2} + \ldots + \frac{1}{(n + p)^2} \leqslant $$ $$\leqslant \frac{1}{n(n + 1)} + \ldots + \frac{1}{(n + p — 1)(n + p)} =$$ $$= \frac{1}{n} — \frac{1}{n + 1} + \ldots + \frac{1}{n + p — 1} — \frac{1}{n + p} =$$ $$= \frac{1}{n} — \frac{1}{n + p} \leqslant \frac{1}{n} < \varepsilon,$$ если только $n \geqslant N = [\frac{1}{\varepsilon}] + 1$. Этим самым доказано, что данная последовательность фундаментальна.

Пример 3. Доказать, что последовательность $x_n = \frac{a_1}{1^2} + \frac{a_2}{2^2} + \ldots + \frac{a_n}{n^2},$ где $|a_n| \leqslant 2$ для всех $n$ натуральных, сходится, с помощью критерия Коши.

Решение

Для натуральных $n$ и $p$ $$|x_{n + p} — x_n| = \frac{|a_{n + 1}|}{(n + 1)^2} + \ldots + \frac{|a_{n + p}|}{(n + p)^2} \leqslant $$ $$\leqslant \frac{2}{(n + 1)^2} + \ldots + \frac{2}{(n + p)^2} \leqslant $$ $$\leqslant \frac{2}{(n + 1)n} + \ldots + \frac{2}{(n + p)(n + p — 1)} =$$ $$= \frac{2}{n} — \frac{2}{n + 1} + \ldots + \frac{2}{n + p — 1} — \frac{2}{n + p} =$$ $$= \frac{2}{n} — \frac{2}{n + p} \leqslant \frac{2}{n} < \varepsilon$$ если только $n \geqslant N = [\frac{2}{\varepsilon}] + 1$. таким образом доказано, что последовательность фундаментальна, а следовательно она сходится.

Упражнение. Покажите, что условие $\lim\limits_{n \to \infty}(x_{n+p} — x_n) = 0$, справедливое при любом натуральном $p$, не влечет фундаментальность последовательности $\{x_n\}$

Литература

Критерий Коши

Тест по теме: «Фундаментальные последовательности. Критерий Коши сходимости числовой последовательности.»


Таблица лучших: Критерий Коши

максимум из 5 баллов
Место Имя Записано Баллы Результат
Таблица загружается
Нет данных

Фундаментальные последовательности и их свойства

Определение

Последовательность [latex] \left \{ x_{n} \right \} [/latex] называют фундаментальной, если она удовлетворяет условию Коши: для каждого [latex]\varepsilon > 0[/latex] существует такое натуральное число [latex] n_{0} [/latex], что для любого [latex]n \geq n_{0}[/latex] и любого [latex]m \geq n_{0}[/latex] справедливо неравенство [latex]\left | x_{n} — x_{m} \right | < \varepsilon[/latex]. Кратко это условие можно записать так: [latex]\forall \varepsilon > 0[/latex]  [latex]\exists n_{0}\in \mathbb{N} :[/latex] [latex]\forall n, m \geq n_{0} :[/latex] [latex]\left | x_{n} — x_{m} \right | < \varepsilon[/latex].

Дадим эквивалентное определение. Последовательность [latex]\left \{ x_{n} \right \}[/latex] называют фундаментальной, если для каждого [latex]\varepsilon > 0[/latex] существует такое натуральное число [latex]n_{0}[/latex], что для любого [latex]n\geq n_{0}[/latex] и для любого натурального [latex]p[/latex] справедливо неравенство [latex]\left | x_{n+p} — x_{n} \right | < \varepsilon[/latex]. Кратко это условие можно записать так: [latex]\forall \varepsilon > 0[/latex]  [latex]\exists n_{0} :[/latex] [latex]\forall n\geq n_{0}[/latex] [latex]\forall p\in \mathbb{N} :[/latex] [latex]\left | x_{n+p} — x_{n} \right | < \varepsilon[/latex].

Докажем, что фундаментальная последовательность является ограниченной. Пусть [latex]\varepsilon = 1[/latex], тогда согласно условию Коши найдется номер $latex n_{0} $ такой, что для всех [latex] n \geq n_{0} [/latex] и для всех [latex]m \geq n_{0}[/latex] выполняется неравенство [latex]\left | x_{n} — x_{m} \right | < 1[/latex], и, в частности, [latex]\left | x_{n} — x_{n_{0}} \right | < 1[/latex]. Так как [latex]\left | x_n \right | = \left | (x_{n}-x_{n_{0}}) + x_{n_{0}} \right |[/latex]   [latex]\leq \left | x_{n_{0}} \right | + \left | x_{n} — x_{n_{0}} \right | [/latex] [latex]< \left | x_{n_{0}} \right | +1[/latex] для  всех [latex] n \geq n_{0} [/latex], то при всех [latex] n \in \mathbb{N}[/latex] справедливо неравенство [latex] \left | x_{n} \right | \le C[/latex], где [latex] C=\max(\left | x_{1} \right | ,\ldots, \left | x_{n_{0}-1} \right | , \left | x_{n_{0}} \right | +1) [/latex]. Это означает, что [latex] \left \{ x_{n} \right \} [/latex] — ограниченная последовательность.

Необходимое и достаточное условие сходимости последовательности

Теорема (критерий Коши)

Для того чтобы последовательность имела конечный предел, необходимо и достаточно, чтобы она была фундаментальной.

Необходимость

Пусть последовательность имеет конечный предел. Положим его равным [latex]a[/latex]. По определению предела [latex] \forall \varepsilon > 0 [/latex]  [latex] \exists n_{0}[/latex] такое, что [latex] \forall k \geq n_{0}[/latex] и выполняется неравенство [latex] \left | x_{k} — a \right | < [/latex] [latex]\frac{\varepsilon}{2} [/latex]. Пусть [latex] k=n[/latex], тогда [latex] \left | x_{n} — a \right | < \frac{\varepsilon}{2} [/latex]. Пусть [latex]k=m[/latex], тогда [latex] \left | x_{m} — a \right | < \frac{\varepsilon}{2} [/latex]. В силу неравенства для модуля суммы (разности), получаем [latex] \left | x_{n}-x_{m} \right |=\left | (x_{n}-a) — (x_{m}-a) \right | [/latex][latex]\leq \left | x_{n}-a \right | + \left | x_{m} — a \right |[/latex] [latex]< \frac{\varepsilon}{2} + \frac{\varepsilon}{2} = \varepsilon [/latex]. Следовательно, для любого [latex] n \geq n_{0}[/latex] и для любого [latex] m \geq n_{0}[/latex] выполняется неравенство [latex] \left | x_{n}-x_{m} \right | < \varepsilon[/latex], т. е. выполняется условие Коши.

Достаточность

Пусть  [latex] \left \{ x_{n} \right \} [/latex]- фундаментальная последовательность. Докажем, что она имеет конечный предел. По определению фундаментальной последовательности [latex] \forall \varepsilon > 0[/latex]  [latex] \exists n_{\varepsilon} :[/latex] [latex] \forall n \geq n_{\varepsilon} [/latex]    [latex] \forall m \geq n_{\varepsilon}[/latex] выполняется неравенство [latex] \left | x_{n} — x_{m} \right | < \frac{\varepsilon}{2}[/latex]. Так как фундаментальная последовательность [latex] \left \{ x_{n} \right \} [/latex] является ограниченной, то, по теореме Больцано-Вейерштрасса, она содержит сходящуюся подпоследовательность [latex] \left \{ x_{n_{k}} \right \} [/latex]. Пусть ее предел равен $latex a $, т. е. [latex] \lim\limits_{ k \to \infty} x_{n_{k}} = a[/latex]. Покажем, что число [latex]a[/latex] является пределом исходной последовательности [latex] \left \{ x_{n} \right \} [/latex]. По определению предела : [latex] \forall \varepsilon > 0[/latex]  [latex] \exists k_{\varepsilon} :[/latex] [latex] \forall k \geq k_{\varepsilon} [/latex] [latex]\rightarrow[/latex] [latex] \left | x_{n_{k}} — a \right | < \frac{\varepsilon}{2}[/latex]. Пусть [latex] N_{\varepsilon} = \max( n_{\varepsilon}, k_{\varepsilon} )[/latex]. Фиксируем  номер [latex] n_{k} \geq N_{\varepsilon}[/latex] (такой номер найдется, так как [latex] n_{k} \to \infty[/latex] при [latex] k \to \infty[/latex] ). Тогда при [latex] m=n_{k}[/latex] и при всех [latex] n \geq N_{\varepsilon}[/latex]  выполняется неравенство [latex] \left | x_{n} — x_{n_{k}} \right | < \frac{\varepsilon}{2}[/latex]. Из этого следует, что при всех  [latex] n \geq N_{\varepsilon}[/latex] справедливо неравенство: [latex] \left | x_{n}-a \right | = \left | (x_{n}-x_{n_{k}}) + (x_{n_{k}}-a) \right | [/latex] [latex]\leq \left | x_{n}-x_{n_{k}}\right | + \left | x_{n_{k}} — a \right | [/latex] [latex]< \frac{\varepsilon}{2} + \frac{\varepsilon}{2} = \varepsilon [/latex] т. е. [latex] \lim\limits_{n \to \infty} x_{n} = a[/latex].

Пример

Доказать, что последовательность [latex]x_{n}=1+ \frac{1}{2} +…+\frac{1}{n}[/latex] расходится.

Спойлер

Последовательность  [latex]\left \{ x_{n} \right \}[/latex] расходится, если не выполняется условие Коши, т.е. [latex]\exists \varepsilon_{0} > 0[/latex]  [latex]\forall k \in \mathbb{N} :[/latex] [latex]\exists n \geq k[/latex]    [latex]\exists m \geq k[/latex], что выполняется неравенство [latex]\left | x_{n} — x_{m} \right | \geq \varepsilon_{0}[/latex]. Пусть задано любое [latex]k \in \mathbb{N}[/latex], положим [latex]n=2k[/latex], [latex]m=k[/latex]. Тогда [latex] \left | x_{n} — x_{m} \right | = \left | x_{2k} — x_{k} \right | = \frac{1}{k+1} + \frac{1}{k+2}+…+ \frac{1}{2k} \geq \frac{1}{2k} k = \frac{1}{2} [/latex]. Таким образом, отрицание условия Коши выполняется при  [latex]\varepsilon_{0} = \frac{1}{2}[/latex], и, в силу критерия Коши, последовательность  [latex] \left \{ x_{n} \right \} [/latex]  расходится.

[свернуть]

Литература

фундаментальные последовательности

Тест на тему «фундаментальные последовательности»:

Фундаментальные последовательности

Фундаментальные последовательности

Последовательность $latex \{x_n\}$ называется фундаментальной, если она удовлетворяет условию Коши:
$latex \forall \varepsilon > 0\ \exists N_\varepsilon :\forall n\geq N_\varepsilon\ \forall p\geq N_\varepsilon\ |x_{n+p}-x_n|\leq \varepsilon\ |x_{n+p}-x_n|\rightarrow 0$
Определение сходимости последовательности и фундаментальности эквивалентны.

Примеры:
Фундаментальными последовательностями являются:

  • $latex \{x_n\}=\frac{\sin\alpha}{2} + \frac{\sin2\alpha}{2^2} + … + \frac{\sin n\alpha}{2^n} $ (можно доказать, используя критерий Коши)
    Спойлер

    $latex |x_{n+p}-x_n|=|\frac{\sin\alpha}{2} + \frac{\sin2\alpha}{2^2} + … + \frac{\sin n\alpha}{2^n}+…+$ $latex +\frac{\sin(n+p)\alpha}{2^{n+p}} — (\frac{\sin\alpha}{2} + \frac{\sin2\alpha}{2^2} + … + \frac{\sin n\alpha}{2^n})|= $$latex |\frac{\sin(n+1)\alpha}{2^{n+1}} + \frac{\sin(n+2)\alpha}{2^{n+2}} + … + \frac{\sin(n+p)\alpha}{2^{n+p}}|\le$ $latex \le |\frac{\sin(n+1)\alpha}{2^{n+1}}| + |\frac{\sin(n+2)\alpha}{2^{n+2}}| + … + |\frac{\sin(n+p)\alpha}{2^{n+p}}|\le$$latex \frac{1}{2^{n+1}} + \frac{1}{2^{n+2}} + … + \frac{1}{2^{n+p}} = $$latex \frac{\frac{1}{2^{n+1}}}{1-\frac{1}{2}}=\frac{2}{2^{n+1}}=\frac{1}{2^n}<\varepsilon \Rightarrow 2^n > \frac{1}{\varepsilon} \Rightarrow n > \log_2\frac{1}{\varepsilon} \Rightarrow n_0=\log_2(\frac{1}{\varepsilon}) + 1$ — таким образом, получили необходимый номер элемента последовательности для каждого $latex \varepsilon$, а значит, последовательность является фундаментальной.

    [свернуть]
  • $latex \{x_n\}=\{1 , \frac{1}{2} , \frac{1}{3} , … , \frac{1}{n}\}$
    Спойлер

    Примем $latex \frac{1}{N_\varepsilon}<\varepsilon$, тогда: $latex |x_{n+p}-x_n|=|\frac{1}{n+p}-\frac{1}{n}|=\frac{1}{n}-\frac{1}{n+p}<\frac{1}{n}<\frac{1}{N_\varepsilon}<\varepsilon$ — таким образом, получили необходимый номер элемента последовательности для каждого $latex \varepsilon$, а значит, последовательность является фундаментальной. 

    [свернуть]
  • $latex \{x_n\}=\frac{3n}{n+1}$
    Спойлер

    $latex |x_{n+p}-x_n|=|\frac{3(n+p)}{n+p+1}-\frac{3n}{n+1}|=|\frac{3n^2+3mn+3n+3m-3n^2-3mn-3n}{(n+p+1)(n+1)}|=$ $latex =|\frac{3m}{(n+p+1)(n+1)}|<\frac{3}{n+1}<\frac{3}{n}$ — таким образом, приняв искомое $latex N_\varepsilon > \frac{3}{\varepsilon}$, получим необходимый номер элемента последовательности для каждого $latex \varepsilon$, а значит, последовательность является фундаментальной.

    [свернуть]

Литература: 

Критерий Коши сходимости последовательности

Критерий Коши сходимости последовательности

Для того, чтобы последовательность имела конечный предел, необходимо и достаточно, чтобы она была фундаментальной.

Доказательство

Необходимость:

Пусть последовательность имеет конечный предел. Докажем, что она является фундаментальной.
Пусть $latex \exists \lim\limits_{n\to\infty}{x_n}=a$ по определению предела последовательности: $latex \forall \varepsilon >0 \ \exists N_\varepsilon :\forall p\geq N_\varepsilon\ |x_p-a|< \varepsilon $

Поскольку $latex \varepsilon $ произвольное, то мы можем взять вместо него, к примеру, $latex \frac{\varepsilon }{2}$:
$latex p=n > N_\varepsilon\ \Bigl|x_n-a\Bigl|<\frac{\varepsilon }{2} $
$latex p=m > N_\varepsilon\ \Bigl|x_m-a\Bigl|<\frac{\varepsilon }{2} $
$latex \Bigl|x_n-x_m\Bigl|=\Bigl|(x_n-a)+(a-x_m)\Bigl|\leq\underset{\underset{\frac{\varepsilon}{2}}{\leq}}{{\underbrace{\Bigl|x_n-a\Bigl|}}} + \underset{\underset{\frac{\varepsilon}{2}}{\leq}}{{\underbrace{\Bigl|x_m-a\Bigl|}}}< \varepsilon $
То есть: $latex \Bigl|x_n-x_m\Bigl| < \varepsilon $, а значит, $latex \{x_n\}_{n=1}^{\infty}$ —   фундаментальная по определению.
Необходимость доказана.

Достаточность:

Пусть $latex \{x_n\}_{n=1}^{\infty}$ — фундаментальная последовательность. Докажем, что она имеет конечный предел. Сначала покажем, что $latex \{{x_n\}}^{\infty}_{n=1}$ — ограничена.
Поскольку $latex \{x_n\}_{n=1}^{\infty}$ — фундаментальная последовательность, то по определению фундаментальной последовательности:
$latex \forall\varepsilon > 0 \ \exists N_\varepsilon :\forall\ n > N_\varepsilon $ и $latex \forall\ m >N_\varepsilon $ $latex |x_n-x_m| < \varepsilon$

Так как $latex \varepsilon $ произвольное, то возьмем $latex \varepsilon=1 : $

$latex \Bigl|x_n\Bigl|=\Bigl|(x_n-x_{N\epsilon})+x_{N\epsilon}\Bigl| \leq\underset{\underset{1}{\leq}}{{\underbrace{\Bigl|x_n-x_{N\epsilon}\Bigl|}}}+\Bigl|x_{N\epsilon}\Bigl|\leq 1+ \Bigl|x_{N\epsilon}\Bigl| $
$latex \forall n \geq N_\varepsilon: |x_n|<(1+|x_{N\epsilon}|)=const=C $ $latex \Bigl|x_n\Bigl|\leq C $
$latex C=\max\{1+\Bigl|x_{N\epsilon}\Bigl|;\Bigl|x_1\Bigl|,\Bigl|x_2\Bigl|,…,\Bigl|x_{N\varepsilon-1}\Bigl|\} \Rightarrow $
$latex \Rightarrow \forall n \epsilon \mathbb{N} : \Bigl|x_n\Bigl|\leq C \Rightarrow $
$latex \{x_n\}_{n=1}^{\infty} $ — ограничена.

По теореме Больцано-Вейерштрасса последовательность $latex \{x_n\}_{n=1}^{\infty}$ имеет сходящуюся подпоследовательность $latex \{{x_{n_k}\}}^{\infty}_{k=1}$

Пусть $latex \lim\limits_{k\rightarrow\infty}{x_{n_k}}=a$, покажем, что число $a$ и будет пределом всей последовательности $latex \{{x_n\}}^{\infty}_{n=1}$:
Поскольку $latex \{x_n\}_{n=1}^{\infty}$ фундаментальная:
$latex \forall \varepsilon>0\ \exists n_\varepsilon : \forall n,m > n_\varepsilon $ $latex |x_n-x_m| <\frac{\varepsilon}{2} $

Так как $latex \{x_{n_k}\}_{k=1}^{\infty}$ сходящаяся:
$latex \lim\limits_{k\rightarrow\infty}{x_{n_k}}=a : \forall \varepsilon>0\ \exists k_\varepsilon :\forall n_k \geq n_{k_\varepsilon} $
$latex |x_{n_k}-a|<\frac{\varepsilon}{2} $
$latex \forall \varepsilon>0 : |x_n-a|=|(x_n-x_{n_k})+(x_{n_k}-a)|\leq |x_n-x_{n_k}|+|x_{n_k}-a|<\varepsilon$
Возьмём $latex N_\varepsilon = \max\{n_\varepsilon, n_{k_\varepsilon}\} $, тогда:$latex \forall \varepsilon >0\ \exists\ N_\varepsilon : \forall n\geq N_\varepsilon : |x_n-a|<\frac{\varepsilon}{2}+\frac{\varepsilon}{2}=\varepsilon $

Достаточность доказана.

Пример 1

Докажем, что последовательность $latex x_N=1+\frac{1}{2}+\frac{1}{3}+…+\frac{1}{N} $ не является фундаментальной.

Спойлер

Покажем, что $latex x_N $ расходящаяся :
Рассмотрим последовательность $latex x_{2N}=1+\frac{1}{2}+\frac{1}{3}+…+\frac{1}{N}+\frac{1}{N+1}+…+\frac{1}{2N} $
Доказательство проведем методом от противного. Предположим, что наша последовательность фундаментальная, тогда по определению фундаментальной последовательности:
$latex \forall \varepsilon >0\ \exists N_\varepsilon :\forall n\geq N_\varepsilon\ m\geq N_\varepsilon\ |x_n-x_m|<\varepsilon\ |x_n-x_m|\rightarrow 0 $ поскольку n и m любые, то возьмём $latex n=N\ m=2N\
\Bigl|x_m-x_n\Bigl|=\Bigl|x_{2N}-x_N\Bigl|=\Bigl|\frac{1}{N+1}+…+\frac{1}{2N}\Bigl|$
таких слагаемых будет N штук, из всех слагаемых $latex \frac{1}{2N} $ — наименьшее.
Можно сказать, что сумма будет больше, чем сумма N наименьших слагаемых, то есть:
$latex |\frac{1}{N+1}+…+\frac{1}{2N}|\geq \frac{1}{2N}*N=\frac{1}{2}$, а значит последовательность не является фундаментальной.
Мы пришли к противоречию.

[свернуть]

Пример 2

Доказать, что последовательность, заданная общим членом $latex x_n=\frac{3n}{n+1}$ фундаментальная.

Спойлер

Доказательство проводится методом подведения под определение. Покажем, что наша последовательность удовлетворяет условию Коши.
Найдём модуль разности между $latex x_n $ и $latex x_{n+m} $
$latex |x_{n+m}-x_n|=\Bigl|\frac{3(n+m)}{n+m+1}-\frac{3n}{n+1}\Bigl|$ $latex =\Bigl|\frac{3n^2+3mn+3n+3m-3n^2-3mn-3n)}{(n+m+1)(n+1)}\Bigl| $
$latex =\Bigl| \frac{3m}{(n+m+1)*(n+1)} \Bigl| $ $latex < \frac{3}{n+1} < \frac{3}{n} $
Если для любого $latex \varepsilon>0$ положить $latex N > \frac{3}{\varepsilon}$, то $latex \forall n > N $ и $latex \forall m\in\mathbb{N}\Rightarrow $ $latex \Bigl|x_{n+m}-x_n\Bigl| < \varepsilon $
Итак, взятая последовательность удовлетворяет критерию Коши, поэтому она сходится (имеет предел).
Причём $latex \lim\limits_{n\to\infty}{x_n}=\lim\limits_{n\to\infty}\ \frac{3}{1+\frac{1}{n}}=3$

[свернуть]

Список литературы:

Тест на тему: Критерий Коши сходимости последовательности

Тест на проверку знаний по данной теме